1988 AIME Problems/Problem 9
Contents
Problem
Find the smallest positive integer whose cube ends in  .
.
Solution 1
A little bit of checking tells us that the units digit must be 2. Now our cube must be in the form of  ; using the binomial theorem gives us
; using the binomial theorem gives us  . Since we are looking for the tens digit,
. Since we are looking for the tens digit,  we get
 we get  . This is true if the tens digit is either
. This is true if the tens digit is either  or
 or  . Casework:
. Casework:
 : Then our cube must be in the form of : Then our cube must be in the form of . Hence the lowest possible value for the hundreds digit is . Hence the lowest possible value for the hundreds digit is , and so , and so is a valid solution. is a valid solution.
 : Then our cube is : Then our cube is . The lowest possible value for the hundreds digit is . The lowest possible value for the hundreds digit is , and we get , and we get . Hence, since . Hence, since , the answer is , the answer is 
Solution 2
 and
 and  .
.
 due to the last digit of
 due to the last digit of  . Let
. Let  . By expanding,
. By expanding,  .
. 
By looking at the last digit again, we see  , so we let
, so we let  where
 where  . Plugging this in to
. Plugging this in to  gives
 gives  . Obviously,
. Obviously,  , so we let
, so we let  where
 where  can be any non-negative integer.
 can be any non-negative integer. 
Therefore,  .
.  must also be a multiple of
 must also be a multiple of  , so
, so  must be even.
 must be even.  . Therefore,
. Therefore,  , where
, where  is any non-negative integer. The number
 is any non-negative integer. The number  has form
 has form  . So the minimum
. So the minimum  .
.
Solution 3
Let  . We factor an
. We factor an  out of the right hand side, and we note that
 out of the right hand side, and we note that  must be of the form
 must be of the form  , where
, where  is a positive integer. Then, this becomes
 is a positive integer. Then, this becomes  . Taking mod
. Taking mod  ,
,  , and
, and  , we get
, we get  ,
,  , and
, and  .
. 
We can work our way up, and find that  ,
,  , and finally
, and finally  . This gives us our smallest value,
. This gives us our smallest value,  , so
, so  , as desired. - Spacesam
, as desired. - Spacesam
Solution 4 (Bash)
Let this integer be  Note that
 Note that ![\[x^3 \equiv 888 \pmod{1000} \implies x \equiv 0 \pmod {2}~~ \cap ~~ x \equiv 2 \pmod{5}.\]](http://latex.artofproblemsolving.com/8/0/3/8037e075c1850fc53973f9576cdceda2f30e1eb5.png) We wish to find the residue of
 We wish to find the residue of  mod
 mod  Note that
 Note that ![\[x \equiv 2,7,12,17, \text{ or } 22 \pmod{25}\]](http://latex.artofproblemsolving.com/5/6/d/56d3345f9cd92a83f9bc41b63a605a8f42bc7b61.png) using our congruence in mod
 using our congruence in mod  The residue that works must also satisfy
 The residue that works must also satisfy  from our original congruence. Noting that
 from our original congruence. Noting that  (and bashing out the other residues perhaps but they're not that hard), we find that
 (and bashing out the other residues perhaps but they're not that hard), we find that ![\[x \equiv 17 \pmod{25}.\]](http://latex.artofproblemsolving.com/1/8/a/18aba95bb531022427765fceae9256080bec119c.png) Thus,
 Thus, ![\[x \equiv 17,42,67,92,117 \pmod{125}.\]](http://latex.artofproblemsolving.com/6/8/1/681e83f7e299d143937dad03c76889e366dbada4.png) The residue that works must also satisfy
 The residue that works must also satisfy  from our original congruence. It is easy to memorize that
 from our original congruence. It is easy to memorize that ![\[17^3 \equiv \mathbf{4913} \equiv 38 \pmod{125}.\]](http://latex.artofproblemsolving.com/0/2/8/02839bc43de45a6d2b3e44de558cef9a98813238.png) Also,
 Also, ![\[42^3 \equiv 42^2 \cdot 42 \equiv 1764 \cdot 42 \equiv 14 \cdot 42 \equiv 88 \pmod{125}.\]](http://latex.artofproblemsolving.com/6/d/2/6d22959730c729d3d23cb5522235c6dadd1d96b6.png) Finally,
 Finally, ![\[67^3 \equiv 67^2 \cdot 67 \equiv 4489 \cdot 67 \equiv (-11) \cdot 67 \equiv -737 \equiv 13 \pmod{125},\]](http://latex.artofproblemsolving.com/6/7/7/6770c40bde2b40c13d1054ed75b2f4c7dcfaea96.png) as desired. Thus,
 as desired. Thus,  must satisfy
 must satisfy ![\[x \equiv 0 \pmod{2}~~ \cap ~~x \equiv 67 \pmod{125} \implies x \equiv 192 \pmod{250} \implies x=\boxed{192}.\]](http://latex.artofproblemsolving.com/1/c/6/1c6b488f5383eb1035baf3dcd1d8fa727a2e6cda.png) ~samrocksnature
 ~samrocksnature
solution 5
This number is in the form of  , after binomial expansion, we only want
, after binomial expansion, we only want  . We realize that
. We realize that  are both multiples of
 are both multiples of  , we only need that
, we only need that  , so we write
, so we write  
Then, we write  so
 so  must be a multiple of
 must be a multiple of  at least, so
 at least, so  after checking, when
 after checking, when  . So
. So  , smallest
, smallest  , the number is
, the number is  
~bluesoul
Solution 6 (A bit of brute force using basic knowledge.)
We do know the unit digit has to be 2, So lets consider the number of the form of  .
.
On cubing  , we get a number of the form
, we get a number of the form  where the unit digit of
 where the unit digit of  must be 8, therefore x can be 4 or 9. But for this value of
 must be 8, therefore x can be 4 or 9. But for this value of  the hundreds digit won't be 8.
 the hundreds digit won't be 8.
Thus our number must be of the form  /
 /  . Repeating the above process we get values of
. Repeating the above process we get values of  as 4 and 1 respectively.
 as 4 and 1 respectively.
Therefore the smallest number is 192.~ Dubey619
See also
| 1988 AIME (Problems • Answer Key • Resources) | ||
| Preceded by Problem 8 | Followed by Problem 10 | |
| 1 • 2 • 3 • 4 • 5 • 6 • 7 • 8 • 9 • 10 • 11 • 12 • 13 • 14 • 15 | ||
| All AIME Problems and Solutions | ||
These problems are copyrighted © by the Mathematical Association of America, as part of the American Mathematics Competitions.  
 . Hence the lowest possible value for the hundreds digit is
. Hence the lowest possible value for the hundreds digit is  is a valid solution.
 is a valid solution. . The lowest possible value for the hundreds digit is
. The lowest possible value for the hundreds digit is  , and we get
, and we get  . Hence, since
. Hence, since  , the answer is
, the answer is 
